LSAT and Law School Admissions Forum

Get expert LSAT preparation and law school admissions advice from PowerScore Test Preparation.

 Administrator
PowerScore Staff
  • PowerScore Staff
  • Posts: 8916
  • Joined: Feb 02, 2011
|
#61051
Please post your questions below!
 Krwill
  • Posts: 9
  • Joined: Jul 09, 2018
|
#62057
Hello,

On the actual exam and doing the exam in practice I selected the same incorrect answer D. I left B as a contender but did not select B because it seemed more vague in that it did not mention anything about political pressure or health issues. For this reason D seemed like a stronger answer. Can you please explain why D is incorrect and B is?
 Jay Donnell
PowerScore Staff
  • PowerScore Staff
  • Posts: 144
  • Joined: Jan 09, 2019
|
#62098
Hi Krwill!

To help explain why B is a better answer than D in bringing in a principle to strengthen the argument, we first have to fully break down the argument in the stimulus.


P: Due to political pressures, the Ministry of the Environment's scientific assessments on the ecological impact of industrial activities are often rendered inaccurate

P: The newly formed Ministry of Health will be facing similar political pressures in relation to health issues

___________________________________________________________________________________________

C: The Ministry of Health should not issue scientific assessments that relate to health issues


The structure of the argument relies on two things:

1) At least some similarity between the two Ministries and their respective battles against political pressure

And perhaps most importantly:

2) A connection between the reports being often inaccurate and the judgment that because of that, the reports should not be issued.


B provides a very useful link to help solidify the argument in regards to the second point listed above, with at least a tangential connection to the first point. Based off the similarity between the Ministries and their respective political pressures facing them, it would be reasonable to say that we DON'T have a strong reason to believe that these reports will be accurate. That allows us to then activate the principle, which takes us nicely to the conclusion that states we should not then issue those scientific assessments.


D fails here as a Strengthen—Principle answer because it relies on the contingency of IF the ministries can successfully resist political pressures to modify the contents of the assessments, and the stimulus offers us no reason to believe that the Ministry of Health will be more successful than the Ministry of the Environment in that regard.

I hope that helps clear it up!
 gfrank
  • Posts: 1
  • Joined: Jul 01, 2019
|
#65961
Hey PowerScore,

I also got down to B and D, yet chose D. At the time, I ultimately chose D because I thought the "believe that those assessments are accurate" portion of B was creating a relationship for the release of the assessments with something from outside the stimulus (the belief of their accuracy) and D was more attractive because it was constructed exclusively with elements from the stimulus (the assessments and political pressure). After reviewing the question, I just want to make sure my corrected line of thinking is valid.

If I diagram the key points of the question:

Stimulus: Political Pressure :arrow: NOT Issue Assessment

Answer D: NOT Political Pressure :arrow: Issue Assessment, NOT Issue Assessment :arrow: Political Pressure

Answer B: Issue Assessment :arrow: Believe Accurate, NOT Believe Accurate :arrow: NOT Issue Assessment

I realize now that there is no way to connect the stimulus with D in a way for Political Pressure to result in NOT Issuing the Assessments. However, connecting B with the stimulus is easy if we infer that Political Pressure is likely to cause the ministries to NOT Believe the assessments are Accurate:

Political Pressure :arrow: NOT Believe Accurate :arrow: NOT Issue Assessment

And then I can look at "Political Pressure :arrow: NOT Believe Accurate" and ask myself if this is reasonable. I should decide that that it is and come to the correct answer choice, B.

I realize that there isn't really a question here but I think writing this out helps me (and maybe someone else) and would love for the experts to double check me! Thank you, this forum has been extremely helpful for me!
 Brook Miscoski
PowerScore Staff
  • PowerScore Staff
  • Posts: 418
  • Joined: Sep 13, 2018
|
#66108
gfrank,

The stimulus establishes that political pressure leads to inaccurate assessments. Then, the stimulus concludes that the new Ministry should not issue assessments. You need to bridge the gap from inaccurate assessments to not publishing the assessments.

Stimulus

Premise: P :arrow: -A
Conclusion P :arrow: -P

Needs: -A :arrow: -I

Answer choice B provides I :arrow: A, and the contrapositive is -A :arrow: -I, which is what you need to bridge the gap.

Your diagram is going towards that approach, so you are on the right track.
 theamazingrace
  • Posts: 59
  • Joined: Oct 17, 2020
|
#80848
Hi, I am having a hard time understanding the explantation to this question. I can't see why B is a better answer then D. Accurate assessment are more important than political pressure even though the inaccuracy in the assessments are due to the political pressure?
 Rachael Wilkenfeld
PowerScore Staff
  • PowerScore Staff
  • Posts: 1358
  • Joined: Dec 15, 2011
|
#81317
Hi amazingrace

This question is a Strengthen—Principle, so we start by looking for the conclusion we need to strengthen. Here, it's that the ministry of health should not issue scientific assessments that relate to health issues. We want to strengthen that principle. It's focused on what the health ministry should not do. That kicks out answer choice (D), which would be strengthening a different conclusion about what the health ministry SHOULD do. We don't care about what they should do--we just care about what they shouldn't do. We also don't want to choose a conditional answer unless the sufficient condition has already been met by the stimulus.

In this case, answer choice (B) is correct because it supports the conclusion that the ministry of health shouldn't issue scientific assessments. If they shouldn't issue assessments unless there's strong reason to believe they will be accurate, that supports the conclusion that the ministry of health should not. We don't have strong reason to believe they will be accurate assessments.

Hope that helps!
Rachael
User avatar
 mkarimi73
  • Posts: 73
  • Joined: Aug 18, 2022
|
#97348
Correct me if I am misunderstanding (B). I interpreted (B) as saying "Scientific assessments should not be issued by government ministries unless THEY (i.e. the government ministries) have a strong reason to believe that those assessments are accurate. How is the contrapositive triggered here? "If THEY (the government ministries) do NOT have a very strong reason to believe that those assessments are accurate, then they shouldn't issue the scientific assessments."

Only the Politician, who is the author of the stimulus, said "these assessments are often inaccurate due to political pressures on the ministry." According to him, they are inaccurate. The stimulus provides no information on whether the government ministries consider them accurate/inaccurate.

And I know exactly what you are going to say: "But that's not what (B) says, Mateen. (B) says whether they have a very strong REASON." Give me a break. What if they do have a strong reason? There's no information in the stimulus that triggers the contrapositive of (B). Please explain how I can get in the mind of the test-makers! Thanks.
 Adam Tyson
PowerScore Staff
  • PowerScore Staff
  • Posts: 5153
  • Joined: Apr 14, 2011
|
#97874
The question only asks for something that would help the argument, mkarimi73 , not something that would absolutely prove it beyond any doubt. Sure, we can nitpick and say "maybe those ministries don't have a strong reason to doubt the accuracy," but the author has already given us a reason to believe that there is such a reason, by comparing it to the similar situation with another ministry. Take that as a fact.

Basically, the correct answer here helps, even if you think it is imperfect. None of the other answers helps at all.

Don't fight the test! You can't win that fight!

Get the most out of your LSAT Prep Plus subscription.

Analyze and track your performance with our Testing and Analytics Package.